Tính Bất Khả Quy Của Đa Thức Có Hệ Số Là Số Nguyên

39 211 0
Tính Bất Khả Quy Của Đa Thức Có Hệ Số Là Số Nguyên

Đang tải... (xem toàn văn)

Tài liệu hạn chế xem trước, để xem đầy đủ mời bạn chọn Tải xuống

Thông tin tài liệu

ĐẠI HỌC THÁI NGUYÊN TRƯỜNG ĐẠI HỌC KHOA HỌC  - NGUYỄN HUY QUÝ TÍNH BẤT KHẢ QUY CỦA ĐA THỨC CÓ HỆ SỐ LÀ SỐ NGUYÊN LUẬN VĂN THẠC SĨ TOÁN HỌC THÁI NGUYÊN - 2016 ĐẠI HỌC THÁI NGUYÊN TRƯỜNG ĐẠI HỌC KHOA HỌC  - NGUYỄN HUY QUÝ TÍNH BẤT KHẢ QUY CỦA ĐA THỨC CÓ HỆ SỐ LÀ SỐ NGUYÊN LUẬN VĂN THẠC SĨ TOÁN HỌC Chuyên ngành: Phương pháp Toán sơ cấp Mã số: 60 46 01 13 NGƯỜI HƯỚNG DẪN KHOA HỌC: GS.TSKH Hà Huy Khoái THÁI NGUYÊN - 2016 i Mục lục Lời cảm ơn iii Mở đầu Chương Tiêu chuẩn bất khả quy Eisenstein, Osada ứng dụng 1.1 Khái niệm đa thức bất khả quy 1.1.1 Vành đa thức 1.1.2 Đa thức bất khả quy 1.1.3 Đa thức bất khả quy Q Đa thức bất khả quy với hệ số nguyên 13 1.2.1 Tiêu chuẩn Eisenstein 13 1.2.2 Tiêu chuẩn Osada 15 1.3 Vận dụng Tiêu chuẩn Eisenstein 16 1.4 Vận dụng Tiêu chuẩn Osada 17 1.2 Chương Tiêu chuẩn bất khả quy Ore, Ram Murty, Chahal, Girstmair ứng dụng 18 2.1 Tính bất khả quy giá trị nguyên tố 18 2.1.1 Tiêu chuẩn Ore 19 2.1.2 Các giá trị nguyên tố tính bất khả quy 26 ii 2.2 Tính bất khả quy đồng dư modulo p 28 Kết luận 33 Tài liệu tham khảo 34 iii Lời cảm ơn Luận văn hoàn thành hướng dẫn GS.TSKH Hà Huy Khoái Xin trân trọng gửi đến Thầy lời cảm ơn sâu sắc Tôi xin trân trọng cảm ơn Ban giám hiệu, Khoa Toán - Tin, Phòng Đào tạo trường Đại học Khoa học - Đại học Thái Nguyên Xin kính gửi lời cảm ơn chân thành đến thầy cố giáo tham gia giảng dạy lớp cao học Toán K8B - khóa học 2014 - 2016 Những người tâm huyết nhiệt tình giảng dạy để trang bị cho tơi kiến thức tốn học bản, đồng thời động viên tạo điều kiện thuận lợi giúp tơi hồn thành nhiệm vụ học tập thời gian qua Tơi xin bày tỏ lịng biết ơn đến gia đình, bạn bè, quan nơi tơi công tác động viên, hỗ trợ tạo điều kiện tốt cho suốt thời gian học tập hoàn thành luận văn Mở đầu Nếu số học, số nguyên tố giữ vai trị quan trọng, đại số, đa thức bất khả quy với hệ số nguyên hay hệ số hữu tỷ có vai trị quan trọng khơng kém, đa thức phân tích thành tích đa thức bất khả quy Trên trường phức, đa thức bất khả quy đa thức bậc nhất, trường thực đa thức bất khả quy đa thức bậc bậc hai Trên trường hữu tỷ đa thức bất khả quy không đơn giản Theo bổ đề Gauss đa thức bất khả quy trường hữu tỷ bất khả quy vành số nguyên Do vậy, việc nghiên cứu tính bất khả quy đa thức với hệ số nguyên cần thiết luôn thời Nhà tốn học tiếng L.Kronecker nói " Chúa cho số nguyên, tất lại tác phẩm người." Từ thời học phổ thông, tất quen thuộc với điểm tương đồng tập hợp số nguyên tập hợp đa thức biến Một mô hình dạng thuật tốn Ơ-clit (với phép chia) Mục đích luận văn trình bày cách tổng quan tính bất khả quy đa thức hệ số nguyên trường Q Trong đó, trình bày số khái niệm biết xung quanh khái niệm đa thức bất khả quy, số tiêu chuẩn bất khả quy đa thức số tập vận dụng Với mục đích luận văn chia làm hai chương: Chương Tiêu chuẩn bất khả quy Eisenstein, Osada ứng dụng Trong chương này, trình bày khái niệm vành đa thức, đa thức bất khả quy; đa thức bất khả quy Q; đa thức bất khả quy với hệ số nguyên; số tiêu chuẩn bất khả quy Eisenstein ứng dụng Chương Tiêu chuẩn bất khả quy Ore, Ram Murty, Chahal, Girstmair ứng dụng Mục tiêu chương trình bày số kết tương đối gần Ore; Ram Murty; Chahal; Girstmair; Schur số ứng dụng Thái Nguyên, ngày 25 tháng năm 2016 Tác giả Nguyễn Huy Quý Chương Tiêu chuẩn bất khả quy Eisenstein, Osada ứng dụng 1.1 1.1.1 Khái niệm đa thức bất khả quy Vành đa thức Nhắc lại tập V= ∅ với phép cộng gọi nhóm điều kiện sau thỏa mãn: (i) Phép cộng có tính kết hợp: a + (b + c) = (a + b) + c với a, b, c ∈ V (ii) Tồn phần tử ∈ V cho a + = + a = a với a ∈ V (iii) Mỗi a ∈ V, tồn phần tử đối −a ∈ V cho a+(−a) = (−a)+a = Nếu thêm điều kiện a + b = b + a với a, b ∈ V V gọi nhóm giao hốn Nhóm cộng V trang bị thêm phép tốn nhân gọi vành điều kiện sau thỏa mãn: (i) Phép nhân có tính kết hợp: (ab)c = a(bc) với a, b ∈ V (ii) Tồn phần tử đơn vị 1∈ V cho a1 = 1a = a với a ∈ V (iii) a(b + c) = ab + ac (b + c)a = ba + ca với a, b, c ∈ V Nếu thêm điều kiện ab = ba với a, b ∈ V V vành giao hoán Định nghĩa 1.1.1 Một đa thức biến x với hệ số V tổng hữu hạn f (x) = an xn + an−1 xn−1 + · · · + a1 x + a0 , a0 , a1 , · · · , an ∈ V Ta ∞ viết đa thức dạng f (x) = ∑ xi , = với i=0 ∞ ∞ i=0 i=0 i > n Hai đa thức ∑ xi ∑ bi xi = bi với i Kí hiệu V [x] tập đa thức biến x với hệ số V Cho f (x) = an xn + an−1 xn−1 + · · · + a1 x + a0 ∈ V [x] Ta gọi a0 hệ số tự f (x) Nếu an = n gọi bậc f (x) kí hiệu deg f (x) Trong trường hợp này, an gọi hệ số cao f (x) Nếu an =1 f (x) gọi đa thức dạng chuẩn (monic polynomial) Ta không định nghĩa bậc cho đa thức Nếu f (x) = a ∈ V f (x) gọi đa thức Các đa thức bậc gọi đa thức tuyến tính Định nghĩa 1.1.2 Với hai đa thức f (x) = ∑ xi g(x) = ∑ bi xi V [x], định nghĩa f (x) + g(x) = ∑(ai + bi )xi f (x)g(x) = ∑ ck xk , ck = ∑ b j với k i+ j=k Khi V [x] vành giao hoán với phép cộng phép nhân đa thức Vành V [x] gọi vành đa thức biến x với hệ số V Phần tử không vành đa thức 0, phần tử đơn vị vành đa thức 1.1.2 Đa thức bất khả quy Trước trình bày khái niệm đa thức bất khả quy, xin nhắc lại khái niệm phần tử bất khả quy miền nguyên Cho a, b ∈ V Ta nói a ước b tồn c ∈ V cho b = ac Một ước a b gọi ước thực b không ước a Phần tử p ∈ V gọi phần tử bất khả quy khác 0, khơng khả nghịch khơng có ước thực Từ ta có khái niệm đa thức bất khả quy vành đa thức V [x] Chú ý V [x] miền nguyên Định nghĩa 1.1.3 Cho f (x) ∈ V [x] đa thức khác không khả nghịch Ta nói f(x) bất khả quy V khơng có ước thực Ta nói f(x) khả quy f (x) có ước thực Chú ý tính bất khả quy đa thức phụ thuộc vào vành sở Chẳng hạn, đa thức 2x + bất khả quy trường Q Tuy nhiên 2x + không bất khả quy vành Z đa thức x + ước thực 2x + Tương tự, đa thức x2 + bất khả quy R không bất khả quy C Bổ đề 1.1.4 (i) Đa thức f(x) bất khả quy f(x+a) bất khả quy với a ∈ V Chứng minh Cho a ∈ V Với đa thức h(x) ∈ V [x] ta đặt h1 (x) = h(x − a) Chú ý degh1 (x) = degh(x) Vì f (x + a) = k(x)g(x) phân tích f (x + a) thành tích hai đa thức có bậc thấp f (x) = k1 (x)g1 (x) phân tích f (x) thành tích hai đa thức có bậc thấp Vì f (x) bất khả quy f (x + a) bất khả quy Từ đến hết mục làm việc với đa thức có hệ số trường K Trong trường hợp này, đa thức khác khả nghịch Do ta có kết sau: (ii) Đa thức f(x) với hệ số trường K bất khả quy degf(x)>0 f(x) khơng phân tích thành tích hai đa thức có bậc bé Sau tính bất khả quy đa thức bậc thấp (iii) Trên trường K, phát biểu sau Đa thức bậc bất khả quy Đa thức bậc bậc bất khả quy khơng có nghiệm K Chứng minh Rõ ràng đa thức bậc khơng thể tích hai đa thức bậc thấp hơn, bất khả quy Giả sử f (x) có nghiệm x = a ∈ K Vì deg f (x) > nên theo kết chứng minh ta có f (x) = (x−a)g(x), g(x) ∈ K[x] degg(x)=deg f (x)1≥1 Do f (x) khả quy Ngược lại, giả sử f (x) khả quy Vì f (x) có bậc nên f (x) phân tích thành tích hai đa thức có bậc thấp hơn, hai đa thức phải có bậc Rõ ràng đa thức bậc trường có nghiệm trường đó, f (x) có nghiệm K Chú ý phát biểu (ii) bổ đề không cho trường hợp bậc đa thức lớn Cụ thể, f (x) bậc lớn có nghiệm K 20 suy (bm+1 − b1 )(bm+1 − b2 ) · · · (bm+1 − bm )g(bm+1 ) = −2 Như vậy, hiệu bm+1 − bi nhận giá trị ±2, ±1 Suy m ≤ Mệnh đề 2.1.5 Nếu f (x) có bậc n u( f ) ≤ n với n ≥ Chứng minh Thật vậy, theo Mệnh đề 2.1.4, ta thấy f (X) nhận đồng thời giá trị + giá trị - 1, u( f ) ≤ ≤ n Nếu f (X) lấy giá trị +1, lấy +1 khơng q n số ngun khác nhau, nghiệm đa thức f (X) − bậc n Do đó, u( f ) ≤ n Khi n ≤ 3, từ Mệnh đề 2.1.4 suy u( f ) ≤ ( f nhận giá trị 1, giá tri -1) Do đó, ta có cận tầm thường cho u( f ) hai lần bậc f ; nghĩa là, u( f ) ≤ 2n Mệnh đề 2.1.6 Nếu P( f ) + 2u ≥ n + f (x) bất khả quy Chứng minh Giả sử f (x) = g(x)h(x) g, h ∈ Z[x] n(g) ≥ 1; n(h) ≥ Khơng tính tổng qt ta giả thiết l(g) ≥ l(h) Nhận xét 2.1.7 l(g) + l(h) ≥ P( f ) + 2u − n Đối với m ∈ Z cho f (m) số nguyên tố, có g(m) h(m) phải đơn vị Vì vậy, u(g) + u(h) ≥ P( f ) + 2u Do có l(g) + l(h) = u(g) − n(g) + u(h) − n(h) = u(g) + u(h) − n ≥ P( f ) + 2u − n, nhận xét Vì theo giả thiết P( f ) + 2u ≥ n + 4, có P( f ) + 2u − n ≥ Vì vậy, với điều kiện có l(g) + l(h) ≥ Nếu l(g) > l(h) > theo định nghĩa có g h đa thức béo Theo Mệnh đề 2.1.4 ta có n(g), n(h) ≤ chúng khơng thể có tổng ≥ Do có g(x) béo Do h(x) không béo n ≥ 7, có n(h) ≥ l(h) ≤ Ngồi ra, l(g) + l(h) ≥ 4, có l(g)= u(g) - n(g) ≥ suy u(g) ≥ + n(g), điều khơng thể n(g) ≤ u(g) ≤ 2n(g) 21 Mâu thuẫn chứng tỏ f (x) bất khả quy Tiếp theo định nghĩa khái niệm "độ cao" đa thức f (x): H1 = max | 0≤i≤n−1 H2 = max | 0≤i≤n−1 | an 1/(n−1) | an Bổ đề sau cho ước lượng nghiệm đa thức Bổ đề 2.1.8 Cho f (x) = an xn + · · · + a1 x + a0 ∈ Z[x] có bậc n với ∈ Z, an = ước chung lớn (a0 , a1 , · · · , an ) = Giả sử an−k = với k = 1, 2, · · · , r, ≤ r ≤ n − Nếu α ∈ C nghiệm f (x) 1/(r+1) |α| < H1 + Nhận xét 2.1.9 Nếu r = phát biểu bổ đề trên, có an−1 = Chứng minh Giả sử α ∈ C nghiệm f (x) an−1 = với k = 1, 2, · · · , r, có −an α n = an−r−1 α n−r−1 + + a1 α + a0 =⇒ −α n = a1 a0 an−r−1 n−r−1 α + + α + an an an Vì vậy, n n−r−1 |α| < H1 (|α| + + |α| + 1) = H1 1/(r+1) Nếu |α| < 1, rõ ràng, |α| < H1 |α|n−r − |α| − (1) + với H1 > Nếu |α| > 1, từ (1), có, |α|n (|α| − 1) ≤ H1 (|α|n−r − 1) < H1 |α|n−r =⇒ |α|r (|α| − 1) < H1 22 Từ (|α| − 1)r+1 ≤ |α|r (|α| − 1), Chúng ta có, (|α| − 1)r+1 < H1 1/(r+1) Và đó, |α| < H1 + Bổ đề 2.1.10 Cho f (x) = an xn + · · · + a1 x + a0 ∈ Z[x] có bậc n với ∈ Z, an = ước chung lớn (a0 , a1 , · · · , an ) = Nếu α ∈ C nghiệm f (x), |α| < 2H2 Chứng minh Đặt bi = /an với i = 0, 1, 2, · · · , n − Giả sử c := max 0≤i≤n−1 |bi |1/(n−i) η = α c Để chứng minh bổ đề, cần chứng tỏ |η| < 2, c = H2 Bởi định nghĩa có |ai /an | ≤ cn−i với i = 0, 1, 2, · · · , n − Sau có, an η n + = an = bn−1 n−1 b0 η +···+ n c c n bn−1 n−1 b0 α + n α +···+ n n c c c (an α n + an−1 α n−1 + · · · + a0 ) n c = f (α) = cn Vì an=0, có, ηn + bn−1 n−1 b0 η + · · · + n = c c 23 Vì |bi | ≤ cn−i , có, |η|n ≤ + |η| + |η|2 + · · · + |η|n−1 (2) Nếu |η| ≥ từ bất đẳng thức (2) có, |η|n ≤ |η|n − |η|n < |η| − |η| − từ suy |η| < 2, mâu thuẫn Do |η| < Khi đó, |α| < 2c Do c = H2 , ta có kết Hệ 2.1.11 Nếu P( f ) ≥ n + u ≥ f (x) bất khả quy Chứng minh tiêu chuẩn Ore Nếu P( f ) ≥ n + theo mệnh đề f (x) bất khả quy Vì vậy, cần giả thiết P( f ) = n + Giả sử f (x) = g(x)h(x), g, h ∈ Z[x] bậc dương Theo nhận xét P( f ) = n + 3, rõ ràng l(g) l(h) dương Do bậc f lớn nên hai đa thức g h phải béo hai Khơng tính tổng qt, giả thiết g béo h không béo Vì vậy, l(g) ≥ l(h) ≤ l(g) + l(h) ≤ l(g) Tuy nhiên theo nhận xét trên, biết l(g) + l(h) ≥ P( f ) + 2u − n ≥ n+3−n = Do đó, nhận l(g) ≥ 3, từ suy u(g) ≥ n+3 Ta kết luận u(g) = n + 1, điều mâu thuẫn với u(g) ≥ n + Do f (x) bất khả quy Giả thuyết sau (vẫn mở) cho thấy tiêu chuẩn Ore chặt chẽ Giả thuyết Đối với n≥ tồn đa thức f (x) ∈ Z[x] khả quy P( f ) = n + Khi n = 2, lấy f (x) = x(x − 4) ta có P( f ) = Khi n = 3, xét f (x) = (x − 5)(1 + x)(x − 3) ta có P( f ) = Định nghĩa 2.1.12 P+ ( f ) = #{ f (n) : n ∈ Z, f (n) > số nguyên tố} Rõ ràng P+ ( f ) đếm số lượng giá trị nguyên tố dương mà f nhận số nguyên phân biệt Định lý sau cung cấp tiêu chuẩn bất khả quy tương tự tiêu chuẩn Ore 24 Định lý 2.1.13 [Chen, et al].(i) Nếu f (x) khả quy P+ ( f ) ≤ n Mặt khác, có đa thức khả quy f ∈ Z[x] mà P+ ( f ) = n (ii) Nếu tồn số nguyên m lớn hệ số cao đa thức f (x) cho f (m) số nguyên tố, f (x) bất khả quy Chứng minh Định lý vượt khuôn khổ luận văn Độc giả quan tâm tìm báo Y.G.Chen, G Kun, G Pete, I Z Ruzza, A Timar "Prime values of reducible polynomials, Acta Arithmetica, 104.2 (2002), 117-127 Do f (x) đa thức biết hệ số nên biết hệ số cao f (x): 1/(r+1) Đặt H = H1 + 1, 2H2 , H1 , H2 "độ cao" đa thức f (x) Khi ta có tiêu chuẩn bất khả quy sau liên quan đến đại lượng (xem [3], [4]) Định lý 2.1.14 Nếu f (x) = an xn + · · · + a1 x + a0 ∈ Z[x] tồn số nguyên m ≥ (H + 1) cho f (m) số nguyên tố f (x) bất khả quy Chứng minh Cho f (x) = an xn + · · · + a1 x + a0 ∈ Z[x], cho α ∈ C 1/(r+1) nghiệm f (x), ta có |α| < H1 + |α| < 2H2 Giả sử f (x) khả quy, gọi f (x) = g(x)h(x) g(x), h(x) ∈ Z[x] có bậc dương Do f (m) số nguyên tố với số nguyên m ≥ (H + 1), ta có g(m) h(m) ±1 Khơng tính tổng quát, giả định g(m) =±1 Viết g(x) = c ∏(x − αi ) αi ∈ C nghiệm i g(x) c hệ số cao g Do αi nghiệm f , nên ta có |αi | < H Vì có mâu thuẫn: = |g(m)| = |c| ∏ |m − αi | ≥ ∏(m − αi ) > i i ∏(m − H) ≥ Do f (x) phải bất khả quy i Nhận xét 2.1.15 (i) r = tốt ý nghĩa sau Xét đa thức khả quy f (x) = (x − 9)(x2 + 1) = x3 − 9x2 + x − có H1 = H = H1 + = 10 Mặc dù f (10) = 101 số nguyên tố đa thức khả quy (ii) Ln ln sử dụng định lý để chứng minh tính bất khả quy 25 f (x) khơng phải ln dể dàng Ví dụ 2.1.16 Giá trị nguyên tố đa thức f (x) = x12 + 488669 xuất với x = 616980 có 70 chữ số thập phân Hệ 2.1.17 Cho p số nguyên tố b ≥ số nguyên Giả sử p viết theo số b sau: p = an bn + an−1 bn−1 + · · · + a1 b + a0 ; ∈ {0, 1, 2, · · · , b − 1} , an = an−1 ∈ {0, 1} Khi đa thức f (x) = an xn + an−1 xn−1 + · · · + a1 x + a0 bất khả quy Chứng minh Giả sử an−1 = Khi rõ ràng H1 ≤ b − r = √ Vì H1 ≤ + b − Nếu chứng minh b > H + 1, f (b) số nguyên tố, theo định lý kết luận √ f (x) bất khả quy Vì vậy, cần chứng tỏ b > + b − 1, tất số b ≥ Thật vậy, tính toán trực tiếp cho ta kết √ √ Bây giả sử an−1 = Vì vậy, H2 ≤ b − suy H ≤ b − √ Chỉ cần chứng minh b − > b − b ≥ 6, điều Từ đó, theo định lý chứng minh, f (x) bất khả quy Ví dụ 2.1.18 Số nguyên tố 104728 = 105 + 4.103 + 7.102 + 2.10 + hệ thập phân thông thường Xét đa thức số f (x) = x5 + 4x3 + 7x2 + 2x + √ Rõ ràng H1 = r = Vì vậy, có 10 > H1 + = + = cho f (10) số nguyên tố Theo định lý f (x) bất khả quy Định lý 2.1.19 [Brillhart, Filaseta Odlyzko, RamMurty][5] Cho b ≥ p số nguyên tố viết theo sở b: p = an bn + an−1 bn−1 + · · · + a1 b + a0 Khi đa thức f (x) = an xn + + a1 x + a0 ∈ Z[x] bất khả quy Chứng minh Giả sử đa thức f (x) = g(x)h(x) với g(x) h(x) đa thức khác đa thức Z[x] Do f (b) số nguyên tố, có g(b) h(b) = ±1 Khơng tính tổng qt, giả sử g(b) = ±1 Viết, g(x) = c ∏(x − αi ) αi ∈ C nghiệm g(x) i 26 c hệ số cao g αi nghiệm f ≤ αi ≤ b − có M = b − √ 1+ 1+4(b−1) Vì vậy, ℜ(α1 ) ≤ |αi | < Do đó: = |g(b)| ≥ ∏ |b − αi | i Chú ý rằng: α√ i = Nếu ℜ(α1 ) ≤ |b − αi | > b 1+ 1+4(b−1) Nếu |αi | < thì|αi | < b − b − |αi | > Từ hai trường hợp trên, có |b − αi | > Do đó, thu 1= g(b) >1 Điều vô lý nên f(x) bất khả quy 2.1.2 Các giá trị nguyên tố tính bất khả quy Lưu ý số tự nhiên n, đa thức X n = X(X − 1) · · · (X − n − 1) n(n − 1) · · · có giá trị nguyên tất số ngun, khơng có hệ số nguyên Quan sát đa thức nhận giá trị nguyên là: Bổ đề 2.1.20 Đa thức P nhận giá trị Z Z nếu, P(x) = a0 + a1 X X + · · · + an n với ∈ Z Chứng minh Điều kiện đủ hiển nhiên Với mệnh đề đảo, trước tiên ta lưu ý đa thức viết dạng n giá trị ( khơng phải giá trị ngun) Viết đa thức P theo dạng giả sử P(Z) ⊂ Z, có: P(0) = a0 ∈ Z P(1) = a0 + a1 ∈ Z P(2) = a0 + a1 ··· + a2 ∈ Z Theo suy luận quy nạp, P(m) ∈ Z ∀m, ta có ∈ Z ∀i 27 Hệ 2.1.21 Nếu đa thức P nhận giá trị Z đến Z có bậc n, n!P(X) ∈ Z[X] Bổ đề 2.1.22 Một đa thức nguyên khác P(X) nhận giá trị số nguyên tố Chứng minh Nếu tất giá trị hợp số, khơng có để chứng minh Vì vậy, giả sử P(a) = p, với a số nguyên p số nguyên tố Khi P khác , lim |P(a + np)| = ∞ Vì vậy, n→∞ n đủ lớn |P(a + np)| > p Nhưng P(a + np) ≡ P(a) ≡ mod p, suy P(a + np) hợp số Nhận xét 2.1.23 Có thể có vơ hạn số ngun tố xuất giá trị nguyên đa thức Ví dụ, (a, b) = 1, Định lý tiếng Dirichlet số nguyên tố cấp số cho thấy đa thức aX + b nhận vơ hạn giá trị số ngun tố Nói chung, khó để xác định xem đa thức có nhận vơ hạn giá trị số ngun tố hay khơng Ví dụ, người ta khơng biết liệu X + có biểu diễn vơ hạn số ngun tố hay khơng? Trong thực tế, khơng có đa thức bậc ≥ nhận vô hạn giá trị số nguyên tố Bổ đề 2.1.24 Nếu đa thức P đa thức có giá trị nguyên khác số ước nguyên tố giá trị {P(m)}m ∈ Z, vơ hạn, tức tất số hạng dãy số P(0), P(1) · · · xây dựng từ hữu hạn số nguyên tố Chứng minh Rõ ràng từ nhận xét ta thấy cần chứng minh cho P(X) ∈ n Z[X], ta giả thiết từ sau Bây xét P(X) = ∑ X i i=0 n ≥ Nếu a0 = rõ ràng P(p) ≡ mod p với số nguyên tố p Nếu a0 = 0, ta xét số nguyên t bất kỳ, đa thức: n n i=0 i=0 i i P(a0tX) = ∑ (a0tX)i = a0 + ∑ ai−1 t X = a0 Q(X) Tồn số nguyên tố p cho Q(m) ≡ 0mod p giá trị m số ngun tố p đó, Q nhận giá trị 0, 1, -1 hữu hạn điểm Khi Q(m) ≡ 1modt, có (p,t) = Khi đó, P(a0tm) ≡ 0mod p Khi giá trị t tùy ý, tập hợp p sinh theo cách vô hạn Bổ đề 2.1.25 Cho a1 , · · · , an số nguyên phân biệt Khi đó, P(X) = (X − a1 ) · · · (X − an ) − bất khả quy 28 Chứng minh Giả sử P(X) = f (X) f (X) với deg f ; degg < n Rõ ràng f (ai ) = −g(ai ) = ±1, ∀1 ≤ i ≤ n Bây giờ, f (X) + g(X) đa thức bậc < n mà triệt tiêu n số nguyên khác a1 , · · · , an nên phải Từ ta có đa thức P(X) = − f (X)2 , điều không thể, thấy so sánh hệ số X n Do P(X) bất khả quy Ví dụ 2.1.26 Cho n số lẻ a1 , · · · , an số nguyên khác Chứng minh (X − a1 ) · · · (X − an ) + bất khả quy Chúng ta xem xét tình sau Giả sử p = an a0 biểu diễn số nguyên tố hệ thập phân thông thường, nghĩa p = a0 +10a1 + 100a2 + + 10n an , với ≤ ≤ Khi đó, đa thức a0 + a1 X + · · · + an X n bất khả quy Trên thực tế, điều đúng, tổng quát ta có: Bổ đề 2.1.27 Cho P(X) ∈ Z[X] giả thiết tồn số nguyên n cho (i) Các không điểm P nằm nửa mặt phẳng Re(z) < n − 12 (ii) P(n − 1) = (iii) P(n) số nguyên tố Khi P(X) bất khả quy Chứng minh Giả sử P(X) = f (X)g(X) Z Tất không điểm f (X) nằm Re(z) < n− 21 Do đó, f (n − 21 − t) < f (n − 21 + t) ∀t > Khi f (n − 1) = f (n − 1) số nguyên, có | f (n − 1)| ≥ Như vậy, | f (n)| > | f (n − 1)| ≥ Điều tương tự g(X), ta thấy P(n) có ước thực f (n), g(n) điều mâu thuẫn với giả thiết Do P(X) bất khả quy 2.2 Tính bất khả quy đồng dư modulo p Giả sử p số nguyên tố không ước hệ số lũy thừa cao đa thức f (x) Dễ thấy rằng, f (x) khả quy Z[x] khả quy F p [x], Fp trường hữu hạn có p phần tử Do có tiêu chuẩn bất khả quy sau đây: Giả sử p số nguyên tố không chia hết hệ số lũy thừa cao đa thức f (x) Khi đó, f (x) bất khả quy Fp [x] f (x) bất khả quy Q 29 Định lý đảo sai Tức là, f (x) bất khả quy Q khơng thiết bất khả quy Fp với số nguyên tố p Ví dụ 2.2.1 Nếu f (x) = x4 + bất khả quy Q ( điều chứng minh trực tiếp từ định nghĩa) khả quy Fp với số nguyên tố p Khi p = 2, rõ ràng f (x) = x4 + ≡ (x2 + 1)2 (mod2) khả quy F2 Cho p ≥ số nguyên tố thỏa mãn p2 ≡ 1(mod8) tức 8|(p2 − 1) (x8 − 1)|(x p −1 − 1) suy 2 x(x4 + 1)(x4 − 1)|(x p − x) Khi f (x) nhân tử đa thức x p − x Trường khai triển K đa thức x p − x Fp Fp2 Rõ ràng [K : Fp ] = Nếu f (x) bất khả quy Fp với số nguyên tố p ≥ K = Fp (α), α nghiệm f (x), trường mở rộng không tầm thường Fp số chiều Ngoài ra, f (x) nhân tử x p − x, K1 trường trung gian K Fp Do đó, có:[K1 : Fp ]|[K : Fp ] => 4|2, vơ lý Do f (x) khả quy Fp với tất số nguyên tố p Đối với đa thức nguyên P(X), có nghiệm modulo số ngun bất kỳ, có nghiệm Z Hilbert người nhận rằng, tính bất khả quy đa thức modulo số nguyên tuỳ ý khơng đủ để đảm bảo tính bất khả quy Z Về nghiệm đa thức modulo số nguyên tố, ta có kết tổng quát sau: Bổ đề 2.2.2 Cho p số nguyên tố P(X) ∈ Z[X] có bậc n Giả sử tất hệ số P bội số p Khi đó, số nghiệm mod p phương trình P(X) ≡ mod p nhiều n Chứng minh Chứng minh rõ ràng cách dùng thuật toán chia Z/p Thật vậy, kết tổng quát thuộc loại (có thể chứng minh thuật tốn chia) nói đa thức khác khơng trường tuỳ ý có số nghiệm khơng vượt q bậc Nhận xét 2.2.3 Khi 1, 2, , p − nghiệm X p−1 ≡ mod p, ta có: X p−1 ≡ (X − 1)(X − 2)(X − (p − 1)) mod p Đối với p lẻ, việc đặt X = cho ta định lý Wilson nói (p − 1)! ≡ −1 mod p Lưu ý, trước biết đa thức ngun có nghiệm modulo vơ hạn số 30 nguyên tố Tuy nhiên Hilbert phát đầu tiên, tính khả quy đa thức modulo số ngun khơng suy tính khả quy Z Ví dụ có kết sau: Bổ đề 2.2.4 Cho p, q số nguyên tố lẻ cho ( qp ) = ( qp ) = p ≡ mod Ở ( qp ) biểu thị kí hiệu Legendre, xác định -1 tuỳ thuộc p phương hay khơng theo modulo q Khi đó, đa thức P(X) = (X − p − q)2 − 4pq bất khả quy, khả quy modulo số nguyên √ √ Chứng minh P(X) = X − 2(p + q)X + (p − q)2 = (X − p − q)(X + √ √ √ √ √ √ p + q)(X − p + q)(X + p − q) √ √ √ √ √ Vì p; q; p ± q; pq vơ tỷ, khơng có thừa số tuyến tính thừa số bậc P(X) thuộc Z[X] nghĩa P(X) bất khả quy Lưu ý rằng, cần chứng tỏ tồn phân tích P modulo lũy thừa nguyên tố nào, sử dụng định lý Trung Hoa để có phân tích modulo số nguyên tuỳ ý Bây P(X) viết dạng sau: P(X) = X − 2(p + q)X + (p − q)2 = (X + p − q)2 − 4pX = (X − p + q)2 − 4qX = (X − p − q)2 − 4pq Các đẳng thức thứ hai thứ ba cho thấy P(X) khả quy modulo pn qn tuỳ ý Cũng vậy, p ≡ mod 8; p thặng dư bậc hai theo 2n đẳng thức thứ hai lần cho thấy P(X) hiệu hai số phương 2n đó, khả quy mod 2n Nếu l số nguyên tố = 2, p, q ta P(X) khả quy modulo l n với n Ít ( pl ), ( ql ) ( pq l ) vì, theo cơng thức tích q pq p ký hiệu Legendre, ( pl ).( ql ).( pq l ) = Tuỳ thuộc ( l ), ( l ) ( l ) = 1; đẳng thức thứ hai, thứ ba thứ tư cho thấy P(X) khả quy modulo l n , n Tôi xin kết thức phần với kết Schur mà chứng minh 31 ông bất ngờ tinh tế Định lý 2.2.5 [Schur] Đối với n bất kỳ, đa thức mũ cắt cụt En X = n!(1 + n X + X2! + + Xn! ) bất khả quy Z Để chứng minh định lý này, cần số kiện lý thuyết số, đặc biệt lý thuyết số đại số Trong chứng minh sử dụng kiện số nguyên tố, quan sát Sylvester ( Định lý Sylvester), chứng minh khó khăn Nếu m ≥ r, (m + 1)(m + 2) · · · (m + r) có thừa số nguyên tố p > r Trường hợp đặc biệt m = r gọi định đề Bertrand Chứng minh (định lý Schur.) Giả sử En (X) = f (X)g(X), đa thức nguyên bất khả quy khác Ta viết f (X) = a0 + a1 X + + X r ( rõ ràng lấy hệ số cao f 1) Bây sử dụng quan sát sau đây, mà thân thú vị: Quan sát Mọi số nguyên tố chia hết số hạng tự a0 f bé bậc r f Đề thấy rõ điều này, lưu ý N(α), chuẩn α ( tên gọi tích tất nghiệm đa thức tối tiểu f α) a0 , sai khác dấu Vì vậy, có i-đê-an ngun tố P Ok để (α) = Pk I, (p) = Pl J I, J khơng chia hết cho P k, l ≥ Ở (α) (p) ký hiệu, tương ứng, i-đê-an Ok sinh α p Vì En (α) = 0, ta có: α2 = n! + n!α + n! ! + · · · + α n Chúng ta biết số mũ xác p chia hết n! hn = [n/p] + n/p2 + · · · Như vậy, Ok , i-đê-an (n!) chia hết cho Plhn không chia hết cho lũy thừa cao P Tương tự vậy, ≤ i ≤ n, i-đê-an sinh n!ai /i! chia hết cho Plhn −lhi +ki Từ đẳng thức 32 −n! = n!α + n!α /2! + · · · + α n ; suy lhn − lhi + ki lớn thực lhn , mà số mũ xác P chia hết vế trái Vì vậy, có số i để lhi + ki ≤ Do đó: i ≤ ki ≤ lhi = l [i/p] + i/p2 + · · · < li p−1 Như vậy, p − < l ≤ r tức p ≤ r Điều chứng minh quan sát nêu Để tiếp tục chứng minh, giả thiết bậc r f n/2 Bây sử dụng định lý Sylvester để chọn số nguyên tố q > r chia hết tích số n(n − 1)(n − r + 1) Lưu ý sử dụng định lý số hạng nhỏ n − r + tích r số liên tiếp lớn r r ≤ n/2 Cũng lưu ý rằng, quan sát cho ta thấy q chia hết a0 Bây giờ, ta viết En (X) theo modulo số nguyên tố q Do cách chọn, q chia hết hệ số X i với ≤ i ≤ n − r n−1 n−r+1 X X + · · · + n! (n−r+1)! mod q Vì vậy, f (X)g(X) ≡ X n + n! (n−1)! Viết f (X) = a0 + a1 X + · · · + X r g(X) = b0 + b1 X + · · · + X n−r Đồng dư cho a0 b0 ≡ 0, a0 b1 + a1 b0 ≡ · · · mod q hệ số X n−r f (X)g(X) Do a0 ≡ 0modq ta nhận b0 ≡ b1 ≡ · · · ≡ bn−r ≡ điều bn−r = Định lý Shur chứng minh mod q 33 Kết luận Luận văn trình bày cách tổng quan tính bất khả quy đa thức hệ số ngun trường Q Trong đó, trình bày số khái niệm biết xung quanh khái niệm đa thức bất khả quy, số tiêu chuẩn bất khả quy đa thức số tập vận dụng Các nội dung luận văn là: Nhắc lại số khái niệm biết đa thức bất khả quy; Một số tiêu chuẩn bất khả quy đa thức với hệ số nguyên trường hữu tỷ Q; Một số tập vận dụng; Giới thiệu số kết tính bất khả quy đa thức với hệ số ngun Trong kết luận văn giới thiệu số kết tương đối gần tính bất khả quy đa thức với hệ số nguyên trường Q ứng dụng 34 Tài liệu tham khảo Tiếng Việt [1] Lê Thị Thanh Nhàn (2015), Lý thuyết đa thức, NXB Giáo dục Tiếng Anh [2] H L Dorwart (1935), Irreducibility of Polynomials, Amer Math Monthly, 42, No 6, 369-381 [3] M Filaseta (1982), A further generalization of an irreducibility theorem of A Cohn , Canad J Math, 34, 1390 -1395 [4] M Filaseta (1988), Irreducibility criteria for polynomials with non negative coefficients , Canad J Math, 40, No 2, 339 -351 [5] R Thangadurai (2007), Irreducibility of polynomials whose coefficients are integer, Mathematics Newsletter, Vol 17, pp.29 - 37 ... khái niệm vành đa thức, đa thức bất khả quy; đa thức bất khả quy Q; đa thức bất khả quy với hệ số nguyên; số tiêu chuẩn bất khả quy Eisenstein ứng dụng 2 Chương Tiêu chuẩn bất khả quy Ore, Ram... trường phức, đa thức bất khả quy đa thức bậc nhất, trường thực đa thức bất khả quy đa thức bậc bậc hai Trên trường hữu tỷ đa thức bất khả quy khơng đơn giản Theo bổ đề Gauss đa thức bất khả quy trường... Nếu số học, số nguyên tố giữ vai trị quan trọng, đại số, đa thức bất khả quy với hệ số nguyên hay hệ số hữu tỷ có vai trị quan trọng khơng kém, đa thức phân tích thành tích đa thức bất khả quy

Ngày đăng: 14/07/2020, 11:20

Từ khóa liên quan

Tài liệu cùng người dùng

Tài liệu liên quan